Math, asked by lambasheetal, 6 months ago

The value of (log_(sqrt(2))(cos20^(@))+log_(sqrt(2))(cos40^(@))+log_(sqrt(2))(cos80^(@)))^(2) +3^(log_((tan60^(@)))(sec60^(@))) is equal to 032sqrt(2)​

Attachments:

Answers

Answered by MaheswariS
1

\textbf{To find:}

\textsf{The value of}

\mathsf{\left(\log_{\sqrt2}cos20^\circ+\log_{\sqrt2}cos40^\circ+\log_{\sqrt2}cos80^\circ\right)^2+3^{\log_{tan60^\circ}sec60^\circ}}

\textbf{Solution:}

\textbf{Formula used:}

\boxed{\mathsf{cos(60^\circ-A)\,cosA\,cos(60^\circ+A)=\dfrac{1}{4}cos3A}}

\mathsf{Consider,}

\mathsf{cos20^\circ\,cos40^\circ\,cos80^\circ}

\mathsf{=cos40^\circ\,cos20^\circ\,cos80^\circ}

\mathsf{=cos(60^\circ-20^\circ)\,cos20^\circ\,cos(60^\circ+20^\circ)}

\mathsf{=\dfrac{1}{4}\,cos3(20^\circ)}

\mathsf{=\dfrac{1}{4}\,cos60^{\circ}}

\mathsf{=\dfrac{1}{4}{\times}\dfrac{1}{2}}

\mathsf{=\dfrac{1}{8}}

\mathsf{Consider,}

\mathsf{\left(\log_{\sqrt2}cos20^\circ+\log_{\sqrt2}cos40^\circ+\log_{\sqrt2}cos80^\circ\right)^2+3^{\log_{tan60^\circ}sec60^\circ}}

\textsf{Using product rule of logarithm, we get}

\mathsf{=\left(\log_{\sqrt2}(cos20^\circ\,cos40^\circ\,cos80^\circ)\right)^2+3^{\log_{\sqrt3}2}}

\mathsf{=\left(\log_{\sqrt2}(\frac{1}{8})\right)^2+3^{\log_{\sqrt3}2}}

\mathsf{=\left(\log_{\sqrt2}(\frac{1}{\sqrt2})^6\right)^2+3^{\log_{\sqrt3}2}}

\mathsf{=\left(\log_{\sqrt2}(\sqrt2)^{-6}\right)^2+(\sqrt{3})^2^{\log_{\sqrt3}2}}

\mathsf{=\left(-6\log_{\sqrt2}{\sqrt2}\right)^2+((\sqrt{3})^2)^{\log_{\sqrt3}2}}

\mathsf{Using,}\;\boxed{\bf\,\log_aa=1}\;\boxed{\bf\,a^{\log_aN=N}}

\mathsf{=\left(-6(1)\right)^2+\sqrt{3}^{2\log_{\sqrt3}2}}

\mathsf{=\left(-6\right)^2+\sqrt{3}^{\log_{\sqrt3}2^2}}

\mathsf{=36+\sqrt{3}^{\log_{\sqrt3}4}}

\mathsf{=36+4}

\mathsf{=40}

\implies\boxed{\mathsf{\left(\log_{\sqrt2}cos20^\circ+\log_{\sqrt2}cos40^\circ+\log_{\sqrt2}cos80^\circ\right)^2+3^{\log_{tan60^\circ}sec60^\circ}=40}}

\textbf{Find more:}

Log (6x^2-5x+1) base 1-2x - log (4x^2 -4x+1) base 1-3x = 2​

https://brainly.in/question/16880478

Similar questions